Diracs Quantisierungsregel

Ich erinnere zunächst an die Quantisierungsregel von Dirac, die unter der Hypothese abgeleitet wurde, dass irgendwo eine magnetische Ladung austreten würde: g q 4 π = n 2 mit n eine natürliche Zahl.

Ich frage mich, wie daraus die Quantisierung der elektrischen Ladung abgeleitet werden kann. Die Quantisierung des Produkts g q ist sicherlich nicht genug; was wird noch gefordert?

Die Quantisierung von gq reicht sicherlich aus, denn wenn man q klein macht, steigt die Einheit von g ins Unendliche. Bei beliebig kleinen Ladungen ist der kleinste Ladungsmonopol unendlich geladen. Wenn es einen festen Ladungsmonopol gibt, ist die Ladungseinheit die Inverse der magnetischen Ladung. Was gibt es noch zu sagen?
Ihre Antwort genügt in einem Universum mit nur zwei Teilchen: eine magnetische Ladung g und eine elektrische Ladung q . Aber wenn Sie jetzt ein größeres Universum betrachten, mit anderen Teilchen, habe ich das Gefühl, ja, es gibt noch mehr zu sagen ...
@Isaac: Nirgendwo in meiner Antwort wird davon ausgegangen, dass es nur zwei Arten von Partikeln gibt. Wenn wir andererseits die Ladungsquantisierung über zwei Arten von Partikeln etabliert haben, bedeutet das Vorhandensein von mehr Arten von Partikeln etwas mehr Einschränkungen (schwach, weil es möglicherweise keine neuen Einschränkungen sind), aber zumindest können sie die vorherigen Bedingungen nicht nur rückgängig machen ziehen Sie sie fest. Die Ladungsquantisierung bleibt also bestehen.

Antworten (4)

i) Zunächst einmal die Dirac-Quantisierungsregel

(1) q g 2 π     Z

für magnetische Monopole kann auf die Dirac-Zwanziger-Schwinger-Quantisierungsbedingung verallgemeinert werden

(2) q 1 g 2 q 2 g 1 2 π     Z

für Dyons . (In einem leichten Terminologiemissbrauch werden wir im Folgenden auch rein elektrisch geladene Teilchen und rein magnetische Monopole in die Definition von Dyonen einbeziehen.)

II) Let Γ = { ( q , g ) } bezeichnen den Satz elektrischer und magnetischer Ladungen für Dyonen. Es ist natürlich, daran zu denken Γ als Teilmenge der Ebene R 2 . Die linke Seite von (2) hat eine geometrische Bedeutung als vorzeichenbehafteter Bereich, der von zwei Vektoren aufgespannt wird ( q 1 , g 1 ) und ( q 2 , g 2 ) .

III) Nehmen Sie das jetzt an Γ { ( 0 , 0 ) } ist nicht leer, dh es existiert ein Dyon ( q 1 , g 1 ) ( 0 , 0 ) zunächst. Welche Punkte ( q 2 , g 2 ) Γ von R 2 würde Bedingung (2) nicht widersprechen? Die Antwort ist eine Menge äquidistanter diskreter Linien parallel zum Vektor ( q 1 , g 1 ) .

IV) Nehmen Sie das jetzt an Γ mindestens zwei linear unabhängige Vektoren enthält ( q 1 , g 1 ) und ( q 2 , g 2 ) . Welche Punkte ( q 3 , g 3 ) Γ von R 2 würde Bedingung (2) nicht widersprechen? Die Antwort ist ein diskretes Gitter aus Schnittpunkten, und zwar genau dort, wo sich die entsprechenden zwei Sätze von äquidistanten diskreten parallelen Linien aus Abschnitt III treffen. Mit anderen Worten, die Ladungen werden quantisiert.

V) Als Spezialfall, wenn es mindestens ein rein elektrisch geladenes Teilchen und mindestens einen rein magnetischen Monopol gibt, befinden wir uns in der in Abschnitt IV beschriebenen Situation, und daher müssen die Ladungen quantisiert werden.

Ich denke, das ist eine gültige offene Frage. Wenn sich herausstellen würde, dass es nicht nur eine magnetische Ladung g gibt, sondern ein Kontinuum magnetischer Ladung, dann wäre die Quantisierungsbedingung keine ausreichende Erklärung für e.

Um jedoch tatsächlich zu beweisen, ob ein Kontinuum magnetischer Ladung zu einem Widerspruch führt oder nicht, müsste zumindest ein n-Körper-Problem mit mehreren magnetischen Ladungen gelöst werden, was selbst für Fachleute auf diesem Gebiet keine triviale Angelegenheit ist. (oder zumindest ein 3-Körper-Problem mit 2 magnetischen Ladungen, um zu sehen, ob ein Widerspruch oder eine Bestätigung der Quantisierung entsteht)

Wenn dieses Thema in der Literatur angesprochen wurde, wäre es schön, wenn jemand mit dem Wissen einige Zitate als Referenzmaterial für Interessierte geben könnte.

1) Angenommen, es existiert eine minimale elektrische Ladung ungleich Null, q 0 . Daher ist die minimale magnetische Ladung

g 0 = 2 π q 0 .

2) Zweitens, wenn die Theorie C und CP bewahrt. Dann der Dyon ( q , g 0 ) impliziert automatisch ein konjugiertes dyon ( q , g 0 ) . Anwenden der Dirac-Zwanziger-Bedingung (siehe in der @Qmechanic-Antwort) für diese beiden Dyons

2 q g 0 = 2 π n ,

oder,

q = n 2 q 0 .

Wir haben also zwei Möglichkeiten, die elektrische Ladung q dauert ganzzahlige Vielfache von q 0 , oder nimmt ungerade ganzzahlige Vielfache von q 0 / 2 .

Ich werde versuchen, aus einer rein mathematischen Perspektive zu antworten. Die Quantisierungsregel besagt, dass für alle möglichen q und g , es gibt einige n Z so dass q g = n h .

Betrachten Sie nun die Menge X = { n Z + | q Q + , g G +   s . t .   q g = n h } , wo Q + enthält alle möglichen positiven Ladungen und G + enthält alle möglichen positiven magnetischen Ladungen (von magnetischen Monopolen). Betrachten Sie das minimale Element n 0 im Satz X , dann gibt es einige q 0 und g 0 das befriedigt q 0 g 0 = n 0 h .

Beachten Sie, dass dies oft angenommen wird n 0 = 1 , wird aber in diesem Beweis nicht benötigt.

Betrachten Sie nun einige g g 0 . Dann q 0 g = n h für einige n . Seit n 0 ist minimal, wir haben n > n 0 . Da beide n und n 0 ganze Zahlen sind, haben wir n = p n 0 + r , 0 r < n 0 , p Z + . Wenn r 0 , dann 0 < q 0 ( g p g 0 ) = r h < n h , dem widersprechen n ist minimal. Deswegen r = 0 . und g = p g 0 , Also g wird in der Einheit von quantisiert g 0 . Genauso können wir das beweisen q muss ein Vielfaches von sein q 0 .

Mathematisch gesehen sind die einzigen Annahmen, die oben verwendet werden, dass wenn q ist dann eine gültige Ladung q ist auch eine gültige Gebühr, und das, wenn q 1 und q 2 sind dann gültige Gebühren q 1 + q 2 ist eine gültige Gebühr. Dasselbe für g . Ich denke, diese Annahmen sollten angesichts der physikalischen Natur von ziemlich natürlich sein q und g .